Write a word phrase for each algebraic expression.
13. n + 6
14. 5 - c
15. 11.5 + y
17. 3x + 10
16. x/4 - 17
18. 10x + 7z

Answers

Answer 1

Step-by-step explanation:

6 more than nc is subtracted from 5y more than 11.510 more than thrice of x17 less than x divided by 4The product of 7 and z is added to the product of 10 and x.

Related Questions

1. A helicopter is at a position from two VORS (VHF Omnidirectional
Radio Range, an aircraft navigation system operating in the VHF band -
not covered in chapter) as in the diagram shown below. Given the angles
shown, find the third angle.
Helicopter
74.0°
66.0°
VOR
VOR

Answers

The position of the helicopter and the two VORs forms a triangle and the third angle formed by these three entities is 40 degrees

The diagram is not shown; however, the question can still be answered.

The given angles are:

[tex]\theta_1 = 74.0^o[/tex]

[tex]\theta_2 = 66.0^o[/tex]

Represent the third angle as [tex]\theta_3[/tex]

The helicopter and the 2 VORs form a triangle.

So, we make use of the following theorem to calculate the third angle

[tex]\theta_1 + \theta_2 + \theta_3= 180^o[/tex] ---- sum of angles in a triangle

Substitute known values

[tex]74.0^o + 66.0^o + \theta_3= 180^o[/tex]

[tex]140.0^o + \theta_3= 180^o[/tex]

Collect like terms

[tex]\theta_3= 180 -140.0^o[/tex]

[tex]\theta_3= 40^o[/tex]

Hence, the third angle is 40 degrees.

Learn more about angles in a triangle at:

https://brainly.com/question/14780489

Find the area of the figure. (Sides meet at right angles.)

Answers

Answer:

56

Step-by-step explanation:

A=(3*4)+(4*(4+3+4))=56

Find an upper bound for E(h) the error of the machine approximation of the two-point forward difference formula for the first derivative and then find the h corresponding to the minimum of E(h).

The two-point forward difference formula for f'(x) is:_________

Answers

Answer:

I doubt it is not going to be a great

Please Help!! Whoever helps and gets it correct gets Brainliest and 5 star rating!!

Answers

Answer:

the reasoning states that "all the numbers begin with a 7 or an 8"

however this is not accurate as they can be in different placements

which can make a big difference in the total estimate.

for example:

the number could've been an 8, or an 80

they both begin with an 8

however have totally different values and could have messed up the total estimated number.

hope this helps :D

Seven and one-half foot-pounds of work is required to compress a spring 2 inches from its natural length. Find the work required to compress the spring an additional 3 inch.

Answers

Answer:

Apply Hooke's Law to the integral application for work: W = int_a^b F dx , we get:

W = int_a^b kx dx

W = k * int_a^b x dx

Apply Power rule for integration: int x^n(dx) = x^(n+1)/(n+1)

W = k * x^(1+1)/(1+1)|_a^b

W = k * x^2/2|_a^b

 

From the given work: seven and one-half foot-pounds (7.5 ft-lbs) , note that the units has "ft" instead of inches.   To be consistent, apply the conversion factor: 12 inches = 1 foot then:

 

2 inches = 1/6 ft

 

1/2 or 0.5 inches =1/24 ft

To solve for k, we consider the initial condition of applying 7.5 ft-lbs to compress a spring  2 inches or 1/6 ft from its natural length. Compressing 1/6 ft of it natural length implies the boundary values: a=0 to b=1/6 ft.

Applying  W = k * x^2/2|_a^b , we get:

7.5= k * x^2/2|_0^(1/6)

Apply definite integral formula: F(x)|_a^b = F(b)-F(a) .

7.5 =k [(1/6)^2/2-(0)^2/2]

7.5 = k * [(1/36)/2 -0]

7.5= k *[1/72]

 

k =7.5*72

k =540

 

To solve for the work needed to compress the spring with additional 1/24 ft, we  plug-in: k =540 , a=1/6 , and b = 5/24 on W = k * x^2/2|_a^b .

Note that compressing "additional one-half inches" from its 2 inches compression is the same as to  compress a spring 2.5 inches or 5/24 ft from its natural length.

W= 540 * x^2/2|_((1/6))^((5/24))

W = 540 [ (5/24)^2/2-(1/6)^2/2 ]

W =540 [25/1152- 1/72 ]

W =540[1/128]

W=135/32 or 4.21875 ft-lbs

Step-by-step explanation:

Round 573.073 to the greatest place

Answers

Answer:

574

Step-by-step explanation:

To round a two-digit number to the nearest ten, simply increase it or decrease it to the nearest number that ends in 0: When a number ends in 1, 2, 3, or 4, bring it down; in other words, keep the tens digit the same and turn the ones digit into a 0

Hope this helps <3

if U>T, R>Q, S>T and T>R, which of the following is TRUE?
1. S>Q
2. U > S
3.U > R​
A. 1 only
B. 2 only
C. 1 and 2
D. 2 and 3

Answers

Answer:

C. 1 and 2

Step-by-step explantation:

First, i would order them as U>T, T>R, R>Q, S>T

we can rewrite them as

U>T>R>Q,

now adding S, we get U>S>T>R>Q,

so U>S

We can also rewrite all of them as inequalities:

U-T>0

T-R>0

R-Q>0

S-T>0

Add R-Q and T-R

(R-Q)+(T-R)>0

-Q+T>0

T>Q, but because S>T we can say S>Q

Find 0.2B
B=[50 10
25 15]

Answers

Multiplying a matrix by a scalar results in every entry in a matrix get multiplied by that scalar, as defined,

[tex]a\begin{bmatrix}b&c\\d&e\\\end{bmatrix}=\begin{bmatrix}ab&ac\\ad&ae\\\end{bmatrix}[/tex]

So in our case, ([tex]0.2=\frac{1}{5}[/tex]

[tex]\frac{1}{5}\begin{bmatrix}50&10\\25&15\\\end{bmatrix}=\begin{bmatrix}\frac{50}{5}&\frac{10}{5}\\\frac{25}{5}&\frac{15}{5}\\\end{bmatrix}=\boxed{\begin{bmatrix}10&2\\5&3\\\end{bmatrix}}[/tex]

Hope this helps :)

Paul can install a 300-square-foot hardwood floor in 18 hours. Matt can install the same floor in 22 hours. How long would it take Paul and Matt to install the floor working together?
4 hours
9.9 hours
13.2 hours
30 hours

Answers

Answer:

9.9 hours

Step-by-step explanation:

The formula to determine the time together is

1/a+1/b = 1/c  where a and b are the times alone and c is the time together

1/18 + 1/22 = 1/c

The least common multiply of the denominators is 198c

198c(1/18 + 1/22 = 1/c)

11c+ 9c = 198

20c = 198

Divide by 20

20c/20 =198/20

c =9.9

Answer:

B - 9.9 hrs

Step-by-step explanation:

took the test.

Solve for X and show your work and explain please

Answers

Answer: x = 45

Step-by-step explanation:

Given

(2/3)x + 4 = (4/5)x - 2

Add 2 on both sides

(2/3)x + 4 + 2 = (4/5)x - 2 + 2

(2/3)x + 6 = (4/5)x

Subtract (2/3)x on both sides

(2/3)x + 6 - (2/3)x = (4/5)x - (2/3)x

6 = (12/15)x - (10/15)x

6 = (2/15)x

Divide 2/15 on both sides

6 / (2/15) = (2/15)x / (2/15)

[tex]\boxed{x=45}[/tex]

Hope this helps!! :)

Please let me know if you have any questions

Answer:

x = 45

Step-by-step explanation:

2/3 x + 4 = 4/5x - 2             Add 2 to both sides

2/3 x + 4 + 2 = 4/5x            Combine

2/3x + 6 = 4/5x                   Subtract 2/3 x from both sides.

6 = 4/5x - 2/3 x                  Multiply both sides by 15

6*15 = 4/5 x * 15 - 2/3x * 15

6*15 = 12x - 10x                   Combine the left and right

90 = 2x                               Divide by 2

x = 45

Let's see if it works.

LHS = 2/3 * 45 + 4

LHS = 2*15 + 4

LHS = 30 + 4

LHS = 34

RHS

Right hand side = 4/5 * 45 - 2

RHS = 36 - 2

RHS = 34 which is the same as the LHS

8.9 x 10^3 in standard notation

Answers

Answer:

that is n standard notation mah frand

8.9 × 10^3 being scientific notation of " 8900 "

[tex]\huge\text{Hey there!}[/tex]

[tex]\large\textsf{8.9}\times\large\textsf{10}^\mathsf{3}\\\\\mathsf{10^3}\\\mathsf{= 10\times10\times10}\\\mathsf{= 100\times10}\\\mathsf{= \bf 1,000}\\\\\large\textsf{8.9}\times\large\textsf{1,000}\\\\\large\textsf{= \bf 8,900}\\\\\\\boxed{\boxed{\huge\text{Answer: \boxed{\underline{\underline{\bf 8,900}}}}}}\huge\checkmark[/tex]

[tex]\huge\text{Good luck on your assignment \& enjoy your day!}[/tex]

~[tex]\boxed{\huge\text{}\boxed{\frak{Amphitrite1040:)}}}[/tex]

find the missing side of the triangle

Answers

Answer:

x = 34

Step-by-step explanation:

Pytago:

x[tex]30^{2} + 16^{2} = x^2\\x = \sqrt{30^2 + 16^2} \\x = 34[/tex]

Find an equation of the plane orthogonal to the line
(x,y,z)=(0,9,6)+t(7,−7,−6)

which passes through the point (9, 6, 0).

Give your answer in the form ax+by+cz=d (with a=7).

Answers

The given line is orthogonal to the plane you want to find, so the tangent vector of this line can be used as the normal vector for the plane.

The tangent vector for the line is

d/dt (⟨0, 9, 6⟩ + ⟨7, -7, -6⟩t ) = ⟨7, -7, -6⟩

Then the plane that passes through the origin with this as its normal vector has equation

x, y, z⟩ • ⟨7, -7, -6⟩ = 0

We want the plane to pass through the point (9, 6, 0), so we just translate every vector pointing to the plane itself by adding ⟨9, 6, 0⟩,

(⟨x, y, z⟩ - ⟨9, 6, 0⟩) • ⟨7, -7, -6⟩ = 0

Simplifying this expression and writing it standard form gives

x - 9, y - 6, z⟩ • ⟨7, -7, -6⟩ = 0

7 (x - 9) - 7 (y - 6) - 6z = 0

7x - 63 - 7y + 42 - 6z = 0

7x - 7y - 6z = 21

so that

a = 7, b = -7, c = -6, and d = 21

An equation of the plane orthogonal to the line 7x - 7y - 6z = 21.

The given line is orthogonal to the plane you want to find,

So the tangent vector of this line can be used as

The normal vector for the plane.

The tangent vector for the line is,

What is the tangent vector?

A tangent vector is a vector that is tangent to a curve or surface at a given point.

d/dt (⟨0, 9, 6⟩ + ⟨7, -7, -6⟩t ) = ⟨7, -7, -6⟩

Then the plane that passes through the origin with this as its normal vector has the equation

⟨x, y, z⟩ • ⟨7, -7, -6⟩ = 0

We want the plane to pass through the point (9, 6, 0), so we just

translate every vector pointing to the plane itself by adding ⟨9, 6, 0⟩,

(⟨x, y, z⟩ - ⟨9, 6, 0⟩) • ⟨7, -7, -6⟩ = 0

Simplifying this expression and writing it in standard form gives

⟨x - 9, y - 6, z⟩ • ⟨7, -7, -6⟩ = 0

7 (x - 9) - 7 (y - 6) - 6z = 0

7x - 63 - 7y + 42 - 6z = 0

7x - 7y - 6z = 21

So that, a = 7, b = -7, c = -6, and d = 21.

To learn more about the equation of plane visit:

https://brainly.com/question/1603217

How many edges are there?

Answers

9514 1404 393

Answer:

  24

Step-by-step explanation:

The front face is an 8-sided star, so has 8 edges. We presume the back face is the same, so it also has 8 edges. Each of the front vertices is connected by an edge to each of the corresponding back vertices, so there are 8 more edges connecting front and back.

The total number of edges is 8 + 8 + 8 = 24.

State if the scenario involves a permutation or a combination. Then find the number of possibilities.

A team of 15 basketball players needs to choose two players to refill the water cooler.

Permutation/Combination:

Answer:

Answers

Answer:

Permutation ; 210 ways

Step-by-step explanation:

Permutation and combination methods refers to mathematical solution to finding the number of ways of making selection for a group of objects.

Usually, selection process whereby the order of selection does not matter are being treated using permutation, while those which takes the order of selection into cognizance are calculated using combination.

Here, selecting 2 players from 15 ; since order does not matter, we use permutation ;

Recall :

nPr = n! ÷ (n - r)!

Hence,

15P2 = 15! ÷ (15 - 2)!

15P2 = 15! ÷ 13!

15P2 = (15 * 14) = 210 ways

which of the following illustrates commutative property of addition? 17+4=4+17​

Answers

9514 1404 393

Answer:

  17 +4 = 4 +17

Step-by-step explanation:

The only expression shown here illustrates that property.

Reason Can you subtract a positive integer from a positive integer
and get a negive result? Explain your answer.

Answers

Answer:

No

Step-by-step explanation:

No matter the situation, when you multiply a negative by a negativeyou get a positive and a positive by a positive you get a positive. but if its two different like a negative and a positive then its NEGITIVE.

let's say you have 23 and you're multiplying by 2.

It's always increasing so it doesnt ever reach the negitive numbers.

Find the length of the arc.

A. 539π/12 km
B. 9π/3 km
C. 9π/2 km
D. 18π km

Answers

Answer:

b because it is I found out cus I took test

The length of the arc 9π/2 km.

The answer is option C.9π/2 km.

What is the arc of the circle?

The arc period of a circle can be calculated with the radius and relevant perspective using the arc period method.

  ⇒angle= arc/radius

     ⇒  135°=arc/6km

     ⇒ arc =135°*6km

     ⇒arc=135°*π/180° * 6km

    ⇒arc = 9π/2 km

Learn more about circle here:-https://brainly.com/question/24375372

#SPJ2

Find the slope of the line that goes through the
(2,6) and (-1, -6)

Answers

We can use the formula y2-y1/x2-x1 to get our slope. y2 and x2 are our second y and x coordinates, meanwhile y1 and x1 are our first y and x coordinates. -6-6/-1 -2 is -12/-3. -12/-3 is 4, the slope is 4.
the slope is 1/4 because you use the slope intercept formula

Write the standard form of the equation of the circle with center (8,−1) that passes through the point (6,7)

Answers

Answer:

(x - 8)^2 + (y + 1)^2 = 68

Step-by-step explanation:

The standard form of the equation of the circle with center (8,−1) is :

(x - 8)^2 + (y + 1)^2 = R^2

If the circle passes through the point (6,7) that means that the point (6,7) is a solution of the equation and we can replace (x,y) with (6,7) to find R.

If the mean of a given dataset is
42 and the standard deviation is
4, where will a majority of the
data lie?

Answers

Answer:

A majority of the data will lie between 38 and 46.

Step-by-step explanation:

It can be said that a majority of the data of a distribution lies within 1 standard deviation of the mean.

In this question:

Mean of 42, standard deviation of 4.

42 - 4 = 38

42 + 4 = 46

A majority of the data will lie between 38 and 46.

please solve the question ​

Answers

Answer:

[tex]g(-1) = -1[/tex]

[tex]g(0.75) = 0[/tex]

[tex]g(1)= 1[/tex]

Step-by-step explanation:

Given

See attachment

Solving (a): g(-1)

We make use of:

[tex]g(x) = -1[/tex]

Because: [tex]-1 \le x < 0[/tex] is true for x =-1

Hence:

[tex]g(-1) = -1[/tex]

Solving (b): g(0.75)

We make use of:

[tex]g(x) = 0[/tex]

Because: [tex]0 \le x < 1[/tex] is true for x =0.75

Hence:

[tex]g(0.75) = 0[/tex]

Solving (b): g(1)

We make use of:

[tex]g(x) = 1[/tex]

Because: [tex]1 \le x < 2[/tex] is true for x =1

Hence:

[tex]g(1)= 1[/tex]

Solve for x: 10/3 = x/(−5/2)

Answers

9514 1404 393

Answer:

  x = -25/3

Step-by-step explanation:

Multiply by the inverse of the coefficient of x. Reduce the fraction.

  (-5/2)(10/3) = (-5/2)(x/(-5/2))

  -50/6 = x = -25/3

Answer:

-25/3

Step-by-step explanation:

the other person is also correct. khan said so

Select all sets in which the number - 15 is an element.
A. natural numbers
B. real numbers
C. irrational numbers
D. rational numbers
E. whole numbers
F. integers

Answers

the answer is A. natural numbers

-15 is an element of-

B. real numbers

D. rational numbers

F. integers

What is a number?

A number is a mathematical object used to count, measure, and label. The original examples are natural number 1,2,3,4 and so forth.

Given number is 15.

A. natural numbers

The natural numbers are the set of all the whole numbers excluding zero. They are positive whole number.

Here, -15 is a negative number,

Hence, -15 is an not element of natural number.

B. real numbers

Real numbers are those numbers that has no imaginary part. It also include both rational and irrational number.

Since -15 has no imaginary part

Hence, -15 is an element of real number.

C. irrational numbers

An irrational number is real number that cannot be expressed as a ratio of two integers.

Here -15 can be expressed as ratio of two integers,

Hence,-15 is not an element of irrational number.

D. rational numbers
A rational number is real number that can be expressed as a ratio of two integers.

Here -15 can be expressed as ratio of two integers,

Hence, -15 is an element of rational number.

E. whole numbers

Whole numbers are positive numbers, including zero, without any decimal or fractional parts. Negative numbers are not considered "whole numbers."

Since,Negative numbers are not considered whole numbers

Hence, -15 is not an element of whole number.

F. integers

An integer is a whole number (not a fractional number) that can be positive, negative, or zero. Examples of integers are: -5, 1, 5, 8, 97, and 3,043

Hence, -15 is an element of integers.

Hence, we conclude that,

-15 is an element of-

B. real numbers

D. rational numbers

F. integers

More about classification of number :

https://brainly.com/question/14213693

#SPJ2

Which ratio is equal to 27 : 81?

Answers

3:9 and if you reduce it again, 1:3

Answer:

1:3

Step-by-step explanation:

27 : 81

Divide each side by 27

27/27 : 81/27

1:3

11 divided by 9876



Thank youuuu

Answers

Answer:

770,7272727272727

Step-by-step explanation:

Answer:

8478 divided by 11= 770.7272727

Identify the transformed function that represents f(x) = ln x stretched vertically by a factor of 17, reflected across the x-axis, and shifted by 19 units left.
A. g(x) = −17ln (x + 19)
B. g(x) = 17ln (x − 19)
C. g(x) = 17ln (x + 19)
D. g(x) = −17ln (x − 19)

Answers

Answer:

b

Step-by-step explanation:

ANSWER. EXPLANATION. The given logarithmic function is. The transformation,. stretches the graph of y=f(x) vertically by a factor of c units ...

4 votes

ANSWER[tex]y = - 3 ln(x - 7) [/tex]EXPLANATIONThe given logarithmic function is [tex]f(x) = ln(x) [/tex]The transformation, [tex]y = - cf(x - k)[/tex]stretches

The length of a rectangle is twice its width. If the area of the rectangle is 72in², find its perimeter

Answers

Let breadth be x

Length=2x

[tex]\\ \sf\longmapsto Area=Length\times Breadth[/tex]

[tex]\\ \sf\longmapsto 72=2x(x)[/tex]

[tex]\\ \sf\longmapsto 2x^2=72[/tex]

[tex]\\ \sf\longmapsto x^2=\dfrac{72}{2}[/tex]

[tex]\\ \sf\longmapsto x^2=36[/tex]

[tex]\\ \sf\longmapsto x=\sqrt{36}[/tex]

[tex]\\ \sf\longmapsto x=6[/tex]

Length=6×2=12inBreadth=6in

[tex]\\ \sf\longmapsto Perimeter=2(L+B)[/tex]

[tex]\\ \sf\longmapsto Perimeter=2(12+6)[/tex]

[tex]\\ \sf\longmapsto Perimeter=2(18)[/tex]

[tex]\\ \sf\longmapsto Perimeter=36in[/tex]

Graph y=|x|+5, how does it compare to parent graph y=|x|

Answers

9514 1404 393

Answer:

  it is shifted 5 units upward

Step-by-step explanation:

The y-coordinate is a measure of the distance above the x-axis. When 5 is added to a y-coordinate, the point is shifted 5 units upward.

The function y = |x| +5 adds 5 units to the y-value of every point of the graph of y = |x|. The graph of y=|x|+5 is shifted 5 units upward from the parent graph.

In a class of 70 pupils, 36 like tasty time , 34 like ice-
cream, 6 like both tasty time }
draw a Venn diagram to show the data.
find how
many
like neither tasty time nor ice-cream

Answers

Step-by-step explanation:

I think this might be the correct answer

The number of pupils that like neither tasty-time nor ice cream is 6 if in a class of 70 pupils, 36 like tasty time, 34 like ice cream, 6 like both tasty times.

What is the Venn diagram?

It is defined as the diagram that shows a logical relation between sets.

The Venn diagram consists of circles to show the logical relation.

We have:

In a class of 70 pupils, 36 like tasty time, 34 like ice cream, 6 like both tasty time.

Total = 70 pupils

Number of like tasty time = 36

Number of like ice cream = 34

Number of like both = 6

Let x be the total number of pupils that like neither tasty-time nor ice cream

The number of pupils that like ice cream only =  34 - 6 = 28

The number of pupils that like tasty-time only = 36 - 6 = 30

From the Venn diagram:

28 + 30 + 6 + x = 70

x = 70 - 64

x = 6

Thus, the number of pupils that like neither tasty-time nor ice cream is 6 if in a class of 70 pupils, 36 like tasty time, 34 like ice cream, 6 like both tasty times.

Learn more about the Venn diagram here:

brainly.com/question/1024798

#SPJ2

Other Questions
the process of the natural environment you already know how will you describe a new job in 1000 words help me please to solve this 2 questions pleasee faster.. i will mark you as brainliest What is the surface area of the composite figure? Hi! Does anyone have some SSAT essay examples? Also, what are some good strategies for writing the SSAT essay? Sue has 3 cats. Each cat eats 1 4 of a tin of cat food each day. Sue buys 4 tins of cat food. Has Sue bought enough cat food to feed her cats for 5 days? You must show how you get your answer Decide whether each statement describes the Patricia class or the plebeian class and then drag it to the correct space? A child of divorce may exhibit which of the following behaviorsA. RelievedB. GuiltyC. DepressionD. All apply Guided PracticeUse the vertical motion formula h = 16t2 + vt + c.A child tosses a ball upward with a starting velocity of 10 ft/s from a height of 3 ft. Substitute the values into the vertical motion formula and let h = 0. Use the quadratic formula to solve for t. How long is the ball in the air?A.0.8 sB.0.8 s or 0.2 sC.0.2 s what is the answer to this? The length of a rectangular playing field is 5m longer than its width. If the perimeter of the field is 150m,find it's width can you write to me explaining this please Does the point (0, 0) satisfy the equation y = x2? Express 3.023 in P form where p and q are integers and q= 0 D How does the design demonstrate the ability to analyze, research, and explore a variety of information sources to adequately address how the target market might interpret a designmessage help The points (63, 121), (71, 180), (67, 140), (65, 108), and (72, 165) give the weight in pounds as a function of height in inches for 5 students ina class. Give the points for these students that represent height as a function of weight{(121, 63), (180, 71), (140, 67), (108, 65), (165, 72)}{(121, 140), (180, 71), (140, 67), (108, 65), (165, 72);{(121, 71), (180, 63), (140, 67), (108, 65), (165, 72)} {(63, 121), (71, 180), (67, 140), 65, 108), (72, 165)) Enter the coordinates of the pointon the unit circle at the given angle. 0 Cn 84 bnh phng tr 37 bnh phng chia 47 Help,anyone can help me do quetion,I will mark brainlest. Find the slope, if it exists, of the line containing the points (10,-3) and (10,-8).Select the correct choice below and, if necessary, fill in the answer box to complete your choice.m=